Integral Operator in $L^2$












3














I was trying to do this exercise and I'm wondering if I figured it out well:



I have $mathcal{H} := L^2(0,1)$ and $T$ the operator with integral kernel $K(x,y) = min{x,y}$, $x,y in [0,1]$. I have to show that $T$ is compact and self-adjoint.



To show that is compact I was thinking to say that because $min{x,y} in [0,1]$ then



begin{equation}
dim(operatorname{Im}T) = 1
end{equation}



(The self adjointness I think is trivial..)So T belongs to finite rank operators and so it is compact. (Is this correct?) Then it asks me to find eigenvalues and eigenvectors of $T$ and here I really don't know how to proceed...










share|cite|improve this question
























  • When you claim the dimension is 1, you mean to claim that there is a function $f$ such that for any function $gin L^2$, there is a constant $lambda=lambda(g)$ such that $int_0^1 K(x,y)g(y)dy = lambda f(x)$?
    – Calvin Khor
    2 days ago










  • Actually I meant that $ImT = < 1 >$. But I don't know if it's right..
    – James Arten
    2 days ago












  • So I think in what I wrote, you'd be claiming that the function $f$ is identically 1. No, this is not true, if you take $g=1in L^2$ then $int_0^1 min(x,y) cdot 1 dy $ is not constant in $x$.
    – Calvin Khor
    2 days ago










  • Yea sorry I didn't mean < 1 > because if I consider $int_0^{1} min{x,y}f(y),dy$ it will be a function of $x$. So the $ImT$ will be generated by all possible linear combinations of $x$, is this right?
    – James Arten
    2 days ago






  • 1




    Please observe in desmos.com/calculator that $T1$ is not linear
    – Calvin Khor
    2 days ago


















3














I was trying to do this exercise and I'm wondering if I figured it out well:



I have $mathcal{H} := L^2(0,1)$ and $T$ the operator with integral kernel $K(x,y) = min{x,y}$, $x,y in [0,1]$. I have to show that $T$ is compact and self-adjoint.



To show that is compact I was thinking to say that because $min{x,y} in [0,1]$ then



begin{equation}
dim(operatorname{Im}T) = 1
end{equation}



(The self adjointness I think is trivial..)So T belongs to finite rank operators and so it is compact. (Is this correct?) Then it asks me to find eigenvalues and eigenvectors of $T$ and here I really don't know how to proceed...










share|cite|improve this question
























  • When you claim the dimension is 1, you mean to claim that there is a function $f$ such that for any function $gin L^2$, there is a constant $lambda=lambda(g)$ such that $int_0^1 K(x,y)g(y)dy = lambda f(x)$?
    – Calvin Khor
    2 days ago










  • Actually I meant that $ImT = < 1 >$. But I don't know if it's right..
    – James Arten
    2 days ago












  • So I think in what I wrote, you'd be claiming that the function $f$ is identically 1. No, this is not true, if you take $g=1in L^2$ then $int_0^1 min(x,y) cdot 1 dy $ is not constant in $x$.
    – Calvin Khor
    2 days ago










  • Yea sorry I didn't mean < 1 > because if I consider $int_0^{1} min{x,y}f(y),dy$ it will be a function of $x$. So the $ImT$ will be generated by all possible linear combinations of $x$, is this right?
    – James Arten
    2 days ago






  • 1




    Please observe in desmos.com/calculator that $T1$ is not linear
    – Calvin Khor
    2 days ago
















3












3








3







I was trying to do this exercise and I'm wondering if I figured it out well:



I have $mathcal{H} := L^2(0,1)$ and $T$ the operator with integral kernel $K(x,y) = min{x,y}$, $x,y in [0,1]$. I have to show that $T$ is compact and self-adjoint.



To show that is compact I was thinking to say that because $min{x,y} in [0,1]$ then



begin{equation}
dim(operatorname{Im}T) = 1
end{equation}



(The self adjointness I think is trivial..)So T belongs to finite rank operators and so it is compact. (Is this correct?) Then it asks me to find eigenvalues and eigenvectors of $T$ and here I really don't know how to proceed...










share|cite|improve this question















I was trying to do this exercise and I'm wondering if I figured it out well:



I have $mathcal{H} := L^2(0,1)$ and $T$ the operator with integral kernel $K(x,y) = min{x,y}$, $x,y in [0,1]$. I have to show that $T$ is compact and self-adjoint.



To show that is compact I was thinking to say that because $min{x,y} in [0,1]$ then



begin{equation}
dim(operatorname{Im}T) = 1
end{equation}



(The self adjointness I think is trivial..)So T belongs to finite rank operators and so it is compact. (Is this correct?) Then it asks me to find eigenvalues and eigenvectors of $T$ and here I really don't know how to proceed...







functional-analysis eigenvalues-eigenvectors compact-operators






share|cite|improve this question















share|cite|improve this question













share|cite|improve this question




share|cite|improve this question








edited 2 days ago









Davide Giraudo

125k16150261




125k16150261










asked 2 days ago









James ArtenJames Arten

12911




12911












  • When you claim the dimension is 1, you mean to claim that there is a function $f$ such that for any function $gin L^2$, there is a constant $lambda=lambda(g)$ such that $int_0^1 K(x,y)g(y)dy = lambda f(x)$?
    – Calvin Khor
    2 days ago










  • Actually I meant that $ImT = < 1 >$. But I don't know if it's right..
    – James Arten
    2 days ago












  • So I think in what I wrote, you'd be claiming that the function $f$ is identically 1. No, this is not true, if you take $g=1in L^2$ then $int_0^1 min(x,y) cdot 1 dy $ is not constant in $x$.
    – Calvin Khor
    2 days ago










  • Yea sorry I didn't mean < 1 > because if I consider $int_0^{1} min{x,y}f(y),dy$ it will be a function of $x$. So the $ImT$ will be generated by all possible linear combinations of $x$, is this right?
    – James Arten
    2 days ago






  • 1




    Please observe in desmos.com/calculator that $T1$ is not linear
    – Calvin Khor
    2 days ago




















  • When you claim the dimension is 1, you mean to claim that there is a function $f$ such that for any function $gin L^2$, there is a constant $lambda=lambda(g)$ such that $int_0^1 K(x,y)g(y)dy = lambda f(x)$?
    – Calvin Khor
    2 days ago










  • Actually I meant that $ImT = < 1 >$. But I don't know if it's right..
    – James Arten
    2 days ago












  • So I think in what I wrote, you'd be claiming that the function $f$ is identically 1. No, this is not true, if you take $g=1in L^2$ then $int_0^1 min(x,y) cdot 1 dy $ is not constant in $x$.
    – Calvin Khor
    2 days ago










  • Yea sorry I didn't mean < 1 > because if I consider $int_0^{1} min{x,y}f(y),dy$ it will be a function of $x$. So the $ImT$ will be generated by all possible linear combinations of $x$, is this right?
    – James Arten
    2 days ago






  • 1




    Please observe in desmos.com/calculator that $T1$ is not linear
    – Calvin Khor
    2 days ago


















When you claim the dimension is 1, you mean to claim that there is a function $f$ such that for any function $gin L^2$, there is a constant $lambda=lambda(g)$ such that $int_0^1 K(x,y)g(y)dy = lambda f(x)$?
– Calvin Khor
2 days ago




When you claim the dimension is 1, you mean to claim that there is a function $f$ such that for any function $gin L^2$, there is a constant $lambda=lambda(g)$ such that $int_0^1 K(x,y)g(y)dy = lambda f(x)$?
– Calvin Khor
2 days ago












Actually I meant that $ImT = < 1 >$. But I don't know if it's right..
– James Arten
2 days ago






Actually I meant that $ImT = < 1 >$. But I don't know if it's right..
– James Arten
2 days ago














So I think in what I wrote, you'd be claiming that the function $f$ is identically 1. No, this is not true, if you take $g=1in L^2$ then $int_0^1 min(x,y) cdot 1 dy $ is not constant in $x$.
– Calvin Khor
2 days ago




So I think in what I wrote, you'd be claiming that the function $f$ is identically 1. No, this is not true, if you take $g=1in L^2$ then $int_0^1 min(x,y) cdot 1 dy $ is not constant in $x$.
– Calvin Khor
2 days ago












Yea sorry I didn't mean < 1 > because if I consider $int_0^{1} min{x,y}f(y),dy$ it will be a function of $x$. So the $ImT$ will be generated by all possible linear combinations of $x$, is this right?
– James Arten
2 days ago




Yea sorry I didn't mean < 1 > because if I consider $int_0^{1} min{x,y}f(y),dy$ it will be a function of $x$. So the $ImT$ will be generated by all possible linear combinations of $x$, is this right?
– James Arten
2 days ago




1




1




Please observe in desmos.com/calculator that $T1$ is not linear
– Calvin Khor
2 days ago






Please observe in desmos.com/calculator that $T1$ is not linear
– Calvin Khor
2 days ago












3 Answers
3






active

oldest

votes


















2














We have
$$
int_{(0,1)^2} |k(x,y)|^2 mathsf d(xtimes y) = int_0^1int_0^1 (xwedge y)^2 mathsf dx mathsf dy
leqslant int_0^1int_0^1 mathsf dx mathsf dy = 1 <infty,
$$

so T is a Hilbert-Schmidt operator and hence is compact.






share|cite|improve this answer





















  • Thanks! What about eigenvalues/eigenvectors?
    – James Arten
    2 days ago



















1














$T$ is a Hibert-Schmidt operator because $min{x,y} in L^2([0,1]times[0,1])$. $Tf$ may be written as
begin{align}
Tf & = int_{0}^{1}K(x,y)f(y)dy \
& = int_{0}^{1}min{x,y}f(y)dy \
& = int_0^xyf(y)dy+xint_x^1 f(y)dy
end{align}

If $Tf=lambda f$ for some $fin L^2$ and $lambdainmathbb{C}setminus{0}$, then the above implies that $Tf$ is equal a.e. to a continuous function on $[0,1]$. Hence, $f$ is equal a.e. to a continuous function. So $Tf$ is continuously differentiable. So, assume without loss of generality, that $f$ is continuous. Then $(Tf)(0)=0$ and $(Tf)'$ exists with
$$
lambda f'= (Tf)'=xf(x)-xf(x)+int_x^1f(y)dty=int_x^1 f(y)dt
$$

So $f$ is $C^2$, $f(0)=0$, $f'(1)=0$ and $lambda f''=-f$ for every eigenfunction with eigenvalue $lambdane 0$. The eigenfunctions with non-zero eigenvalues are, therefore, constant multiplies of
$$
f_n = sin(npi x/2),;;; n=1,3,5,7,cdots, \
lambda_n = frac{2}{npi}.
$$



The adjoint of $int_0^x$ is $int_x^1$. And the adjoint of $M_x$ (multiplication by $x$) is $M_x$. So $T$ is selfadjoint because
begin{align}
T &= left(int_0^xright)M_x+M_xleft(int_x^1right)\ &=left(int_0^xright)M_x+M_x^*left(int_0^xright)^* = T^*.end{align}






share|cite|improve this answer





























    0














    Since we have the integral identity
    $$ g:= Tf = int_0^x yf(y) dy + xint_x^1 f(y) dy $$
    Then $gin H^1$, and since $g' = int_x^1 f(y) dy$, actually $g in H^2$, with
    $$ -g'' = f$$
    So we actually have a Poisson equation, but with boundary conditions $g(0)=0,g(1) = int_0^1 yf(y) dy$. By setting $tilde g = g - x int_0^1 yf(y) dy$, we notice that we are equivalently looking for the weak solutions in $H^1_0$ to the Poisson equation with Dirichlet boundary conditions
    $$ -tilde g'' = f text{ on } (0,1),quad tilde g(0)=tilde g(1)=0,quad fin L^2 $$
    So if you already knew that the solution operator $fmapsto tilde g$ for this 1D Poisson equation was compact($H^1_0 subsetsubset L^2)$ and self-adjoint, we're done (the difference $ g-tilde g= xint_0^1 y f(y) dy$ is finite rank and self-adjoint).






    share|cite|improve this answer





















      Your Answer





      StackExchange.ifUsing("editor", function () {
      return StackExchange.using("mathjaxEditing", function () {
      StackExchange.MarkdownEditor.creationCallbacks.add(function (editor, postfix) {
      StackExchange.mathjaxEditing.prepareWmdForMathJax(editor, postfix, [["$", "$"], ["\\(","\\)"]]);
      });
      });
      }, "mathjax-editing");

      StackExchange.ready(function() {
      var channelOptions = {
      tags: "".split(" "),
      id: "69"
      };
      initTagRenderer("".split(" "), "".split(" "), channelOptions);

      StackExchange.using("externalEditor", function() {
      // Have to fire editor after snippets, if snippets enabled
      if (StackExchange.settings.snippets.snippetsEnabled) {
      StackExchange.using("snippets", function() {
      createEditor();
      });
      }
      else {
      createEditor();
      }
      });

      function createEditor() {
      StackExchange.prepareEditor({
      heartbeatType: 'answer',
      autoActivateHeartbeat: false,
      convertImagesToLinks: true,
      noModals: true,
      showLowRepImageUploadWarning: true,
      reputationToPostImages: 10,
      bindNavPrevention: true,
      postfix: "",
      imageUploader: {
      brandingHtml: "Powered by u003ca class="icon-imgur-white" href="https://imgur.com/"u003eu003c/au003e",
      contentPolicyHtml: "User contributions licensed under u003ca href="https://creativecommons.org/licenses/by-sa/3.0/"u003ecc by-sa 3.0 with attribution requiredu003c/au003e u003ca href="https://stackoverflow.com/legal/content-policy"u003e(content policy)u003c/au003e",
      allowUrls: true
      },
      noCode: true, onDemand: true,
      discardSelector: ".discard-answer"
      ,immediatelyShowMarkdownHelp:true
      });


      }
      });














      draft saved

      draft discarded


















      StackExchange.ready(
      function () {
      StackExchange.openid.initPostLogin('.new-post-login', 'https%3a%2f%2fmath.stackexchange.com%2fquestions%2f3063087%2fintegral-operator-in-l2%23new-answer', 'question_page');
      }
      );

      Post as a guest















      Required, but never shown

























      3 Answers
      3






      active

      oldest

      votes








      3 Answers
      3






      active

      oldest

      votes









      active

      oldest

      votes






      active

      oldest

      votes









      2














      We have
      $$
      int_{(0,1)^2} |k(x,y)|^2 mathsf d(xtimes y) = int_0^1int_0^1 (xwedge y)^2 mathsf dx mathsf dy
      leqslant int_0^1int_0^1 mathsf dx mathsf dy = 1 <infty,
      $$

      so T is a Hilbert-Schmidt operator and hence is compact.






      share|cite|improve this answer





















      • Thanks! What about eigenvalues/eigenvectors?
        – James Arten
        2 days ago
















      2














      We have
      $$
      int_{(0,1)^2} |k(x,y)|^2 mathsf d(xtimes y) = int_0^1int_0^1 (xwedge y)^2 mathsf dx mathsf dy
      leqslant int_0^1int_0^1 mathsf dx mathsf dy = 1 <infty,
      $$

      so T is a Hilbert-Schmidt operator and hence is compact.






      share|cite|improve this answer





















      • Thanks! What about eigenvalues/eigenvectors?
        – James Arten
        2 days ago














      2












      2








      2






      We have
      $$
      int_{(0,1)^2} |k(x,y)|^2 mathsf d(xtimes y) = int_0^1int_0^1 (xwedge y)^2 mathsf dx mathsf dy
      leqslant int_0^1int_0^1 mathsf dx mathsf dy = 1 <infty,
      $$

      so T is a Hilbert-Schmidt operator and hence is compact.






      share|cite|improve this answer












      We have
      $$
      int_{(0,1)^2} |k(x,y)|^2 mathsf d(xtimes y) = int_0^1int_0^1 (xwedge y)^2 mathsf dx mathsf dy
      leqslant int_0^1int_0^1 mathsf dx mathsf dy = 1 <infty,
      $$

      so T is a Hilbert-Schmidt operator and hence is compact.







      share|cite|improve this answer












      share|cite|improve this answer



      share|cite|improve this answer










      answered 2 days ago









      Math1000Math1000

      19k31745




      19k31745












      • Thanks! What about eigenvalues/eigenvectors?
        – James Arten
        2 days ago


















      • Thanks! What about eigenvalues/eigenvectors?
        – James Arten
        2 days ago
















      Thanks! What about eigenvalues/eigenvectors?
      – James Arten
      2 days ago




      Thanks! What about eigenvalues/eigenvectors?
      – James Arten
      2 days ago











      1














      $T$ is a Hibert-Schmidt operator because $min{x,y} in L^2([0,1]times[0,1])$. $Tf$ may be written as
      begin{align}
      Tf & = int_{0}^{1}K(x,y)f(y)dy \
      & = int_{0}^{1}min{x,y}f(y)dy \
      & = int_0^xyf(y)dy+xint_x^1 f(y)dy
      end{align}

      If $Tf=lambda f$ for some $fin L^2$ and $lambdainmathbb{C}setminus{0}$, then the above implies that $Tf$ is equal a.e. to a continuous function on $[0,1]$. Hence, $f$ is equal a.e. to a continuous function. So $Tf$ is continuously differentiable. So, assume without loss of generality, that $f$ is continuous. Then $(Tf)(0)=0$ and $(Tf)'$ exists with
      $$
      lambda f'= (Tf)'=xf(x)-xf(x)+int_x^1f(y)dty=int_x^1 f(y)dt
      $$

      So $f$ is $C^2$, $f(0)=0$, $f'(1)=0$ and $lambda f''=-f$ for every eigenfunction with eigenvalue $lambdane 0$. The eigenfunctions with non-zero eigenvalues are, therefore, constant multiplies of
      $$
      f_n = sin(npi x/2),;;; n=1,3,5,7,cdots, \
      lambda_n = frac{2}{npi}.
      $$



      The adjoint of $int_0^x$ is $int_x^1$. And the adjoint of $M_x$ (multiplication by $x$) is $M_x$. So $T$ is selfadjoint because
      begin{align}
      T &= left(int_0^xright)M_x+M_xleft(int_x^1right)\ &=left(int_0^xright)M_x+M_x^*left(int_0^xright)^* = T^*.end{align}






      share|cite|improve this answer


























        1














        $T$ is a Hibert-Schmidt operator because $min{x,y} in L^2([0,1]times[0,1])$. $Tf$ may be written as
        begin{align}
        Tf & = int_{0}^{1}K(x,y)f(y)dy \
        & = int_{0}^{1}min{x,y}f(y)dy \
        & = int_0^xyf(y)dy+xint_x^1 f(y)dy
        end{align}

        If $Tf=lambda f$ for some $fin L^2$ and $lambdainmathbb{C}setminus{0}$, then the above implies that $Tf$ is equal a.e. to a continuous function on $[0,1]$. Hence, $f$ is equal a.e. to a continuous function. So $Tf$ is continuously differentiable. So, assume without loss of generality, that $f$ is continuous. Then $(Tf)(0)=0$ and $(Tf)'$ exists with
        $$
        lambda f'= (Tf)'=xf(x)-xf(x)+int_x^1f(y)dty=int_x^1 f(y)dt
        $$

        So $f$ is $C^2$, $f(0)=0$, $f'(1)=0$ and $lambda f''=-f$ for every eigenfunction with eigenvalue $lambdane 0$. The eigenfunctions with non-zero eigenvalues are, therefore, constant multiplies of
        $$
        f_n = sin(npi x/2),;;; n=1,3,5,7,cdots, \
        lambda_n = frac{2}{npi}.
        $$



        The adjoint of $int_0^x$ is $int_x^1$. And the adjoint of $M_x$ (multiplication by $x$) is $M_x$. So $T$ is selfadjoint because
        begin{align}
        T &= left(int_0^xright)M_x+M_xleft(int_x^1right)\ &=left(int_0^xright)M_x+M_x^*left(int_0^xright)^* = T^*.end{align}






        share|cite|improve this answer
























          1












          1








          1






          $T$ is a Hibert-Schmidt operator because $min{x,y} in L^2([0,1]times[0,1])$. $Tf$ may be written as
          begin{align}
          Tf & = int_{0}^{1}K(x,y)f(y)dy \
          & = int_{0}^{1}min{x,y}f(y)dy \
          & = int_0^xyf(y)dy+xint_x^1 f(y)dy
          end{align}

          If $Tf=lambda f$ for some $fin L^2$ and $lambdainmathbb{C}setminus{0}$, then the above implies that $Tf$ is equal a.e. to a continuous function on $[0,1]$. Hence, $f$ is equal a.e. to a continuous function. So $Tf$ is continuously differentiable. So, assume without loss of generality, that $f$ is continuous. Then $(Tf)(0)=0$ and $(Tf)'$ exists with
          $$
          lambda f'= (Tf)'=xf(x)-xf(x)+int_x^1f(y)dty=int_x^1 f(y)dt
          $$

          So $f$ is $C^2$, $f(0)=0$, $f'(1)=0$ and $lambda f''=-f$ for every eigenfunction with eigenvalue $lambdane 0$. The eigenfunctions with non-zero eigenvalues are, therefore, constant multiplies of
          $$
          f_n = sin(npi x/2),;;; n=1,3,5,7,cdots, \
          lambda_n = frac{2}{npi}.
          $$



          The adjoint of $int_0^x$ is $int_x^1$. And the adjoint of $M_x$ (multiplication by $x$) is $M_x$. So $T$ is selfadjoint because
          begin{align}
          T &= left(int_0^xright)M_x+M_xleft(int_x^1right)\ &=left(int_0^xright)M_x+M_x^*left(int_0^xright)^* = T^*.end{align}






          share|cite|improve this answer












          $T$ is a Hibert-Schmidt operator because $min{x,y} in L^2([0,1]times[0,1])$. $Tf$ may be written as
          begin{align}
          Tf & = int_{0}^{1}K(x,y)f(y)dy \
          & = int_{0}^{1}min{x,y}f(y)dy \
          & = int_0^xyf(y)dy+xint_x^1 f(y)dy
          end{align}

          If $Tf=lambda f$ for some $fin L^2$ and $lambdainmathbb{C}setminus{0}$, then the above implies that $Tf$ is equal a.e. to a continuous function on $[0,1]$. Hence, $f$ is equal a.e. to a continuous function. So $Tf$ is continuously differentiable. So, assume without loss of generality, that $f$ is continuous. Then $(Tf)(0)=0$ and $(Tf)'$ exists with
          $$
          lambda f'= (Tf)'=xf(x)-xf(x)+int_x^1f(y)dty=int_x^1 f(y)dt
          $$

          So $f$ is $C^2$, $f(0)=0$, $f'(1)=0$ and $lambda f''=-f$ for every eigenfunction with eigenvalue $lambdane 0$. The eigenfunctions with non-zero eigenvalues are, therefore, constant multiplies of
          $$
          f_n = sin(npi x/2),;;; n=1,3,5,7,cdots, \
          lambda_n = frac{2}{npi}.
          $$



          The adjoint of $int_0^x$ is $int_x^1$. And the adjoint of $M_x$ (multiplication by $x$) is $M_x$. So $T$ is selfadjoint because
          begin{align}
          T &= left(int_0^xright)M_x+M_xleft(int_x^1right)\ &=left(int_0^xright)M_x+M_x^*left(int_0^xright)^* = T^*.end{align}







          share|cite|improve this answer












          share|cite|improve this answer



          share|cite|improve this answer










          answered 2 days ago









          DisintegratingByPartsDisintegratingByParts

          58.7k42579




          58.7k42579























              0














              Since we have the integral identity
              $$ g:= Tf = int_0^x yf(y) dy + xint_x^1 f(y) dy $$
              Then $gin H^1$, and since $g' = int_x^1 f(y) dy$, actually $g in H^2$, with
              $$ -g'' = f$$
              So we actually have a Poisson equation, but with boundary conditions $g(0)=0,g(1) = int_0^1 yf(y) dy$. By setting $tilde g = g - x int_0^1 yf(y) dy$, we notice that we are equivalently looking for the weak solutions in $H^1_0$ to the Poisson equation with Dirichlet boundary conditions
              $$ -tilde g'' = f text{ on } (0,1),quad tilde g(0)=tilde g(1)=0,quad fin L^2 $$
              So if you already knew that the solution operator $fmapsto tilde g$ for this 1D Poisson equation was compact($H^1_0 subsetsubset L^2)$ and self-adjoint, we're done (the difference $ g-tilde g= xint_0^1 y f(y) dy$ is finite rank and self-adjoint).






              share|cite|improve this answer


























                0














                Since we have the integral identity
                $$ g:= Tf = int_0^x yf(y) dy + xint_x^1 f(y) dy $$
                Then $gin H^1$, and since $g' = int_x^1 f(y) dy$, actually $g in H^2$, with
                $$ -g'' = f$$
                So we actually have a Poisson equation, but with boundary conditions $g(0)=0,g(1) = int_0^1 yf(y) dy$. By setting $tilde g = g - x int_0^1 yf(y) dy$, we notice that we are equivalently looking for the weak solutions in $H^1_0$ to the Poisson equation with Dirichlet boundary conditions
                $$ -tilde g'' = f text{ on } (0,1),quad tilde g(0)=tilde g(1)=0,quad fin L^2 $$
                So if you already knew that the solution operator $fmapsto tilde g$ for this 1D Poisson equation was compact($H^1_0 subsetsubset L^2)$ and self-adjoint, we're done (the difference $ g-tilde g= xint_0^1 y f(y) dy$ is finite rank and self-adjoint).






                share|cite|improve this answer
























                  0












                  0








                  0






                  Since we have the integral identity
                  $$ g:= Tf = int_0^x yf(y) dy + xint_x^1 f(y) dy $$
                  Then $gin H^1$, and since $g' = int_x^1 f(y) dy$, actually $g in H^2$, with
                  $$ -g'' = f$$
                  So we actually have a Poisson equation, but with boundary conditions $g(0)=0,g(1) = int_0^1 yf(y) dy$. By setting $tilde g = g - x int_0^1 yf(y) dy$, we notice that we are equivalently looking for the weak solutions in $H^1_0$ to the Poisson equation with Dirichlet boundary conditions
                  $$ -tilde g'' = f text{ on } (0,1),quad tilde g(0)=tilde g(1)=0,quad fin L^2 $$
                  So if you already knew that the solution operator $fmapsto tilde g$ for this 1D Poisson equation was compact($H^1_0 subsetsubset L^2)$ and self-adjoint, we're done (the difference $ g-tilde g= xint_0^1 y f(y) dy$ is finite rank and self-adjoint).






                  share|cite|improve this answer












                  Since we have the integral identity
                  $$ g:= Tf = int_0^x yf(y) dy + xint_x^1 f(y) dy $$
                  Then $gin H^1$, and since $g' = int_x^1 f(y) dy$, actually $g in H^2$, with
                  $$ -g'' = f$$
                  So we actually have a Poisson equation, but with boundary conditions $g(0)=0,g(1) = int_0^1 yf(y) dy$. By setting $tilde g = g - x int_0^1 yf(y) dy$, we notice that we are equivalently looking for the weak solutions in $H^1_0$ to the Poisson equation with Dirichlet boundary conditions
                  $$ -tilde g'' = f text{ on } (0,1),quad tilde g(0)=tilde g(1)=0,quad fin L^2 $$
                  So if you already knew that the solution operator $fmapsto tilde g$ for this 1D Poisson equation was compact($H^1_0 subsetsubset L^2)$ and self-adjoint, we're done (the difference $ g-tilde g= xint_0^1 y f(y) dy$ is finite rank and self-adjoint).







                  share|cite|improve this answer












                  share|cite|improve this answer



                  share|cite|improve this answer










                  answered 2 days ago









                  Calvin KhorCalvin Khor

                  11.2k21438




                  11.2k21438






























                      draft saved

                      draft discarded




















































                      Thanks for contributing an answer to Mathematics Stack Exchange!


                      • Please be sure to answer the question. Provide details and share your research!

                      But avoid



                      • Asking for help, clarification, or responding to other answers.

                      • Making statements based on opinion; back them up with references or personal experience.


                      Use MathJax to format equations. MathJax reference.


                      To learn more, see our tips on writing great answers.





                      Some of your past answers have not been well-received, and you're in danger of being blocked from answering.


                      Please pay close attention to the following guidance:


                      • Please be sure to answer the question. Provide details and share your research!

                      But avoid



                      • Asking for help, clarification, or responding to other answers.

                      • Making statements based on opinion; back them up with references or personal experience.


                      To learn more, see our tips on writing great answers.




                      draft saved


                      draft discarded














                      StackExchange.ready(
                      function () {
                      StackExchange.openid.initPostLogin('.new-post-login', 'https%3a%2f%2fmath.stackexchange.com%2fquestions%2f3063087%2fintegral-operator-in-l2%23new-answer', 'question_page');
                      }
                      );

                      Post as a guest















                      Required, but never shown





















































                      Required, but never shown














                      Required, but never shown












                      Required, but never shown







                      Required, but never shown

































                      Required, but never shown














                      Required, but never shown












                      Required, but never shown







                      Required, but never shown







                      Popular posts from this blog

                      Mario Kart Wii

                      What does “Dominus providebit” mean?

                      Antonio Litta Visconti Arese